Maximum value of function $f(x)=frac{x^4-x^2}{x^6+2x^3-1}$ when $x >1$












1












$begingroup$


What is the maximum value of the $$f(x)=frac{x^4-x^2}{x^6+2x^3-1}$$ where $x > 1$ .



My try



Image 1



Unable to solve further.










share|cite|improve this question











$endgroup$












  • $begingroup$
    What are $a$ and $b$ on the second page?
    $endgroup$
    – Matti P.
    Jan 16 at 11:43










  • $begingroup$
    Hint: the derivative can be written as $$ f'(x) = frac{-2x(x^2 +1)(x^2-x-1)(x^4 + x^3 - x^2 - x -1)}{(x^6 + 2x^3 -1)^2} $$ Source: wolframalpha.com/input/…
    $endgroup$
    – Matti P.
    Jan 16 at 11:46










  • $begingroup$
    $f(x)$ would be maximum when $1/f(x)$ would be minimum.
    $endgroup$
    – Paras Khosla
    Jan 16 at 14:11
















1












$begingroup$


What is the maximum value of the $$f(x)=frac{x^4-x^2}{x^6+2x^3-1}$$ where $x > 1$ .



My try



Image 1



Unable to solve further.










share|cite|improve this question











$endgroup$












  • $begingroup$
    What are $a$ and $b$ on the second page?
    $endgroup$
    – Matti P.
    Jan 16 at 11:43










  • $begingroup$
    Hint: the derivative can be written as $$ f'(x) = frac{-2x(x^2 +1)(x^2-x-1)(x^4 + x^3 - x^2 - x -1)}{(x^6 + 2x^3 -1)^2} $$ Source: wolframalpha.com/input/…
    $endgroup$
    – Matti P.
    Jan 16 at 11:46










  • $begingroup$
    $f(x)$ would be maximum when $1/f(x)$ would be minimum.
    $endgroup$
    – Paras Khosla
    Jan 16 at 14:11














1












1








1





$begingroup$


What is the maximum value of the $$f(x)=frac{x^4-x^2}{x^6+2x^3-1}$$ where $x > 1$ .



My try



Image 1



Unable to solve further.










share|cite|improve this question











$endgroup$




What is the maximum value of the $$f(x)=frac{x^4-x^2}{x^6+2x^3-1}$$ where $x > 1$ .



My try



Image 1



Unable to solve further.







real-analysis functions optimization maxima-minima a.m.-g.m.-inequality






share|cite|improve this question















share|cite|improve this question













share|cite|improve this question




share|cite|improve this question








edited Jan 16 at 15:48







cattt

















asked Jan 16 at 11:40









catttcattt

242




242












  • $begingroup$
    What are $a$ and $b$ on the second page?
    $endgroup$
    – Matti P.
    Jan 16 at 11:43










  • $begingroup$
    Hint: the derivative can be written as $$ f'(x) = frac{-2x(x^2 +1)(x^2-x-1)(x^4 + x^3 - x^2 - x -1)}{(x^6 + 2x^3 -1)^2} $$ Source: wolframalpha.com/input/…
    $endgroup$
    – Matti P.
    Jan 16 at 11:46










  • $begingroup$
    $f(x)$ would be maximum when $1/f(x)$ would be minimum.
    $endgroup$
    – Paras Khosla
    Jan 16 at 14:11


















  • $begingroup$
    What are $a$ and $b$ on the second page?
    $endgroup$
    – Matti P.
    Jan 16 at 11:43










  • $begingroup$
    Hint: the derivative can be written as $$ f'(x) = frac{-2x(x^2 +1)(x^2-x-1)(x^4 + x^3 - x^2 - x -1)}{(x^6 + 2x^3 -1)^2} $$ Source: wolframalpha.com/input/…
    $endgroup$
    – Matti P.
    Jan 16 at 11:46










  • $begingroup$
    $f(x)$ would be maximum when $1/f(x)$ would be minimum.
    $endgroup$
    – Paras Khosla
    Jan 16 at 14:11
















$begingroup$
What are $a$ and $b$ on the second page?
$endgroup$
– Matti P.
Jan 16 at 11:43




$begingroup$
What are $a$ and $b$ on the second page?
$endgroup$
– Matti P.
Jan 16 at 11:43












$begingroup$
Hint: the derivative can be written as $$ f'(x) = frac{-2x(x^2 +1)(x^2-x-1)(x^4 + x^3 - x^2 - x -1)}{(x^6 + 2x^3 -1)^2} $$ Source: wolframalpha.com/input/…
$endgroup$
– Matti P.
Jan 16 at 11:46




$begingroup$
Hint: the derivative can be written as $$ f'(x) = frac{-2x(x^2 +1)(x^2-x-1)(x^4 + x^3 - x^2 - x -1)}{(x^6 + 2x^3 -1)^2} $$ Source: wolframalpha.com/input/…
$endgroup$
– Matti P.
Jan 16 at 11:46












$begingroup$
$f(x)$ would be maximum when $1/f(x)$ would be minimum.
$endgroup$
– Paras Khosla
Jan 16 at 14:11




$begingroup$
$f(x)$ would be maximum when $1/f(x)$ would be minimum.
$endgroup$
– Paras Khosla
Jan 16 at 14:11










1 Answer
1






active

oldest

votes


















2












$begingroup$

Let $x-frac{1}{x}=t$.



Thus, $t>0$ and by AM-GM we obtain:
$$frac{x^4-x^2}{x^6+2x^3-1}=frac{x-frac{1}{x}}{x^2-frac{1}{x^3}+2}=frac{t}{t^3+3t+2}=$$
$$=frac{1}{t^2+frac{2}{t}+3}leqfrac{1}{3sqrt[3]{t^2left(frac{1}{t}right)^2}+3}=frac{1}{6}.$$
The equality occurs for $t=1$, which says that we got a maximal value.






share|cite|improve this answer









$endgroup$













    Your Answer





    StackExchange.ifUsing("editor", function () {
    return StackExchange.using("mathjaxEditing", function () {
    StackExchange.MarkdownEditor.creationCallbacks.add(function (editor, postfix) {
    StackExchange.mathjaxEditing.prepareWmdForMathJax(editor, postfix, [["$", "$"], ["\\(","\\)"]]);
    });
    });
    }, "mathjax-editing");

    StackExchange.ready(function() {
    var channelOptions = {
    tags: "".split(" "),
    id: "69"
    };
    initTagRenderer("".split(" "), "".split(" "), channelOptions);

    StackExchange.using("externalEditor", function() {
    // Have to fire editor after snippets, if snippets enabled
    if (StackExchange.settings.snippets.snippetsEnabled) {
    StackExchange.using("snippets", function() {
    createEditor();
    });
    }
    else {
    createEditor();
    }
    });

    function createEditor() {
    StackExchange.prepareEditor({
    heartbeatType: 'answer',
    autoActivateHeartbeat: false,
    convertImagesToLinks: true,
    noModals: true,
    showLowRepImageUploadWarning: true,
    reputationToPostImages: 10,
    bindNavPrevention: true,
    postfix: "",
    imageUploader: {
    brandingHtml: "Powered by u003ca class="icon-imgur-white" href="https://imgur.com/"u003eu003c/au003e",
    contentPolicyHtml: "User contributions licensed under u003ca href="https://creativecommons.org/licenses/by-sa/3.0/"u003ecc by-sa 3.0 with attribution requiredu003c/au003e u003ca href="https://stackoverflow.com/legal/content-policy"u003e(content policy)u003c/au003e",
    allowUrls: true
    },
    noCode: true, onDemand: true,
    discardSelector: ".discard-answer"
    ,immediatelyShowMarkdownHelp:true
    });


    }
    });














    draft saved

    draft discarded


















    StackExchange.ready(
    function () {
    StackExchange.openid.initPostLogin('.new-post-login', 'https%3a%2f%2fmath.stackexchange.com%2fquestions%2f3075636%2fmaximum-value-of-function-fx-fracx4-x2x62x3-1-when-x-1%23new-answer', 'question_page');
    }
    );

    Post as a guest















    Required, but never shown

























    1 Answer
    1






    active

    oldest

    votes








    1 Answer
    1






    active

    oldest

    votes









    active

    oldest

    votes






    active

    oldest

    votes









    2












    $begingroup$

    Let $x-frac{1}{x}=t$.



    Thus, $t>0$ and by AM-GM we obtain:
    $$frac{x^4-x^2}{x^6+2x^3-1}=frac{x-frac{1}{x}}{x^2-frac{1}{x^3}+2}=frac{t}{t^3+3t+2}=$$
    $$=frac{1}{t^2+frac{2}{t}+3}leqfrac{1}{3sqrt[3]{t^2left(frac{1}{t}right)^2}+3}=frac{1}{6}.$$
    The equality occurs for $t=1$, which says that we got a maximal value.






    share|cite|improve this answer









    $endgroup$


















      2












      $begingroup$

      Let $x-frac{1}{x}=t$.



      Thus, $t>0$ and by AM-GM we obtain:
      $$frac{x^4-x^2}{x^6+2x^3-1}=frac{x-frac{1}{x}}{x^2-frac{1}{x^3}+2}=frac{t}{t^3+3t+2}=$$
      $$=frac{1}{t^2+frac{2}{t}+3}leqfrac{1}{3sqrt[3]{t^2left(frac{1}{t}right)^2}+3}=frac{1}{6}.$$
      The equality occurs for $t=1$, which says that we got a maximal value.






      share|cite|improve this answer









      $endgroup$
















        2












        2








        2





        $begingroup$

        Let $x-frac{1}{x}=t$.



        Thus, $t>0$ and by AM-GM we obtain:
        $$frac{x^4-x^2}{x^6+2x^3-1}=frac{x-frac{1}{x}}{x^2-frac{1}{x^3}+2}=frac{t}{t^3+3t+2}=$$
        $$=frac{1}{t^2+frac{2}{t}+3}leqfrac{1}{3sqrt[3]{t^2left(frac{1}{t}right)^2}+3}=frac{1}{6}.$$
        The equality occurs for $t=1$, which says that we got a maximal value.






        share|cite|improve this answer









        $endgroup$



        Let $x-frac{1}{x}=t$.



        Thus, $t>0$ and by AM-GM we obtain:
        $$frac{x^4-x^2}{x^6+2x^3-1}=frac{x-frac{1}{x}}{x^2-frac{1}{x^3}+2}=frac{t}{t^3+3t+2}=$$
        $$=frac{1}{t^2+frac{2}{t}+3}leqfrac{1}{3sqrt[3]{t^2left(frac{1}{t}right)^2}+3}=frac{1}{6}.$$
        The equality occurs for $t=1$, which says that we got a maximal value.







        share|cite|improve this answer












        share|cite|improve this answer



        share|cite|improve this answer










        answered Jan 16 at 13:49









        Michael RozenbergMichael Rozenberg

        104k1892197




        104k1892197






























            draft saved

            draft discarded




















































            Thanks for contributing an answer to Mathematics Stack Exchange!


            • Please be sure to answer the question. Provide details and share your research!

            But avoid



            • Asking for help, clarification, or responding to other answers.

            • Making statements based on opinion; back them up with references or personal experience.


            Use MathJax to format equations. MathJax reference.


            To learn more, see our tips on writing great answers.




            draft saved


            draft discarded














            StackExchange.ready(
            function () {
            StackExchange.openid.initPostLogin('.new-post-login', 'https%3a%2f%2fmath.stackexchange.com%2fquestions%2f3075636%2fmaximum-value-of-function-fx-fracx4-x2x62x3-1-when-x-1%23new-answer', 'question_page');
            }
            );

            Post as a guest















            Required, but never shown





















































            Required, but never shown














            Required, but never shown












            Required, but never shown







            Required, but never shown

































            Required, but never shown














            Required, but never shown












            Required, but never shown







            Required, but never shown







            Popular posts from this blog

            Can a sorcerer learn a 5th-level spell early by creating spell slots using the Font of Magic feature?

            Does disintegrating a polymorphed enemy still kill it after the 2018 errata?

            A Topological Invariant for $pi_3(U(n))$